GMAT Prep Practice Test 1 order question

This topic has expert replies
Newbie | Next Rank: 10 Posts
Posts: 8
Joined: Tue Jul 24, 2007 8:24 pm

GMAT Prep Practice Test 1 order question

by davidnodine » Mon Aug 27, 2007 4:56 pm
If x is positive, which of the following could be the correct ordering of 1/x,2x, and x^2

I. x^2<2x<1/x
II. x^2<1/x<2x
III. 2x< x^2<1/x

a. None
b. I only
c. III only
d. I and II
e I, II and III

I understand how one works with fractions but I'm confused that the answer is D I and II. I'm sure its something simple that i'm over thinking. Thanks

Senior | Next Rank: 100 Posts
Posts: 98
Joined: Mon Jun 11, 2007 10:32 pm
Location: Mumbai
davidnodine wrote:If x is positive, which of the following could be the correct ordering of 1/x,2x, and x^2

I. x^2<2x<1/x
II. x^2<1/x<2x
III. 2x< x^2<1/x

a. None
b. I only
c. III only
d. I and II
e I, II and III

I understand how one works with fractions but I'm confused that the answer is D I and II. I'm sure its something simple that i'm over thinking. Thanks
Are you sure II = x^2<1/x<2x ?

Coz if you take a fraction like 3/2,
x^2 = 9/4
1/x = 2/3
2x = 3

So i'm getting the expression as 1/x<x^2<2x.

Master | Next Rank: 500 Posts
Posts: 214
Joined: Fri Jul 20, 2007 1:35 am
Thanked: 3 times

Re: GMAT Prep Practice Test 1 order question

by beny » Tue Aug 28, 2007 1:53 am
ratindasgupta wrote:
davidnodine wrote:If x is positive, which of the following could be the correct ordering of 1/x,2x, and x^2

I. x^2<2x<1/x
II. x^2<1/x<2x
III. 2x< x^2<1/x

a. None
b. I only
c. III only
d. I and II
e I, II and III

I understand how one works with fractions but I'm confused that the answer is D I and II. I'm sure its something simple that i'm over thinking. Thanks
Are you sure II = x^2<1/x<2x ?

Coz if you take a fraction like 3/2,
x^2 = 9/4
1/x = 2/3
2x = 3

So i'm getting the expression as 1/x<x^2<2x.
Just because one example works, doesn't mean he typed the problem incorrectly...

x=.9 would satisfy criterion II.

Newbie | Next Rank: 10 Posts
Posts: 8
Joined: Tue Jul 24, 2007 8:24 pm

by davidnodine » Tue Aug 28, 2007 2:23 pm
It is written correctly. Is there any way to look at this question without plugging in numbers?

Master | Next Rank: 500 Posts
Posts: 447
Joined: Sun Apr 19, 2009 9:08 pm
Location: Kolkata,India
Thanked: 7 times
GMAT Score:670

by uptowngirl92 » Sat Nov 07, 2009 4:39 am
Source of the question is: GMAT Prep Test 1

If x is positive which of the following could be correct ordering of 1/x, 2x, x^2.
1) x^2 < 2x < 1/x

2) x^2 < 1/x < 2x

3) 2x < x^2 < 1/x

Choices are:

1) None
2) 1 Only
3) 3 Only
4) I and 2 only
5) 1, 2, 3

How to proceed without plugging in?I plugged in in the exam and got only choice 1 as correct and hence marke db which is wrong:(

Master | Next Rank: 500 Posts
Posts: 199
Joined: Sat Oct 24, 2009 4:43 pm
Thanked: 22 times
GMAT Score:710

by palvarez » Sat Nov 07, 2009 11:20 am
uptowngirl92 wrote:Source of the question is: GMAT Prep Test 1

If x is positive which of the following could be correct ordering of 1/x, 2x, x^2.
1) x^2 < 2x < 1/x

2) x^2 < 1/x < 2x

3) 2x < x^2 < 1/x

Choices are:

1) None
2) 1 Only
3) 3 Only
4) I and 2 only
5) 1, 2, 3

How to proceed without plugging in?I plugged in in the exam and got only choice 1 as correct and hence marke db which is wrong:(
Hybrid approach to make things smooth

"Could be true" Find one positive instance for each case. Proving that there are no positive instances requires algebraic way.

"must be true" Find one negative instance for each case. Proving that there are all positive instances requires algebraic way.

Lets focus on algebraic way.

1. x^2 < 2x < 1/x
x^2 - 2x < 0 --> (0,2)
2x < 1/x
x^2 - (1/2) < 0 --> (-1/sqrt(2), 1/sqrt(2))

Combining together: (0, 1/sqrt(2))

The domain of this inequality is the above.

2. x^2 < 1/x < 2x

x^3 < 1
domain: (-inf, 1)

and x^2 > 1/2
domain: (-inf, 1/sqrt(2)) U (1/sqrt(2), +inf)

Find the intersection of both domains: (-inf, 1/sqrt(2). Note that x is positive.
(0, 1/sqrt(2)) is the domain.

3. 2x < x^2 < 1/x

x^2 - 2x > 0
domain: (-inf, 0) U (2, +inf)


x^3 < 1
domain: (-inf, 1)

Intersection: (-inf, 0)

But x is positive.


Some key things to remember:

(x-a)(x-b) > 0, (-inf, a) U (b, +inf), assumin a < b
(x-a)(x-b) < 0 (a, b), assumin a < b
(x^3-a^3) > 0 (a, +inf)
(x^3+a^3) < 0 (-inf, a)

Being cool and taking the restctions (like x being +ve) into account help your test.

Smart numbers help you to get rid of some choices, thereby ending up with fewer choices. There, focus on algebraic approach.

Senior | Next Rank: 100 Posts
Posts: 39
Joined: Sat Mar 07, 2009 8:20 pm

by sme0928 » Wed Dec 02, 2009 12:08 pm
Since we know X is positive, cant we multiply by X to get rid of the fraction and then compare?

GMAT Instructor
Posts: 1302
Joined: Mon Oct 19, 2009 2:13 pm
Location: Toronto
Thanked: 539 times
Followed by:164 members
GMAT Score:800

by Testluv » Wed Dec 02, 2009 8:30 pm
x = 1/2 satisfies I

x = 3/4 satisfies II

If x = 3/4, then x^2 = 9/16, and 1/x = 4/3, and 2x = 3/2

9/16 < 4/3 < 3/2

So, both I and II are possible orders. If x is positive, however, it is impossible that x^2 < 1/x while simultaneously 2x < x^2.

Choose D.
Kaplan Teacher in Toronto

Legendary Member
Posts: 869
Joined: Wed Aug 26, 2009 3:49 pm
Location: California
Thanked: 13 times
Followed by:3 members

by heshamelaziry » Fri Dec 04, 2009 5:23 pm
Testluv wrote:x = 1/2 satisfies I

x = 3/4 satisfies II

If x = 3/4, then x^2 = 9/16, and 1/x = 4/3, and 2x = 3/2

9/16 < 4/3 < 3/2

So, both I and II are possible orders. If x is positive, however, it is impossible that x^2 < 1/x while simultaneously 2x < x^2.

Choose D.
But in order for a choice such as II to be correct, the choice has to be consistent with any value for x. Clearly, x=1 will destroy the equation and x = 3/2 will destroy it. How can we chose based on one value only. I haven't seen this yet ?

GMAT Instructor
Posts: 1302
Joined: Mon Oct 19, 2009 2:13 pm
Location: Toronto
Thanked: 539 times
Followed by:164 members
GMAT Score:800

by Testluv » Fri Dec 04, 2009 8:21 pm
heshamelaziry wrote:
Testluv wrote:x = 1/2 satisfies I

x = 3/4 satisfies II

If x = 3/4, then x^2 = 9/16, and 1/x = 4/3, and 2x = 3/2

9/16 < 4/3 < 3/2

So, both I and II are possible orders. If x is positive, however, it is impossible that x^2 < 1/x while simultaneously 2x < x^2.

Choose D.
But in order for a choice such as II to be correct, the choice has to be consistent with any value for x. Clearly, x=1 will destroy the equation and x = 3/2 will destroy it. How can we chose based on one value only. I haven't seen this yet ?
No, it doesn't have to be consistent with any value of x. The question asks which of them COULD be a possible correct ordering. So, we only need to find a value of x that will work. As soon, as we do, we know that that COULD be the possible order.
Kaplan Teacher in Toronto